Movie Screens in Washington County

This topic has expert replies
User avatar
Legendary Member
Posts: 777
Joined: Fri Jan 01, 2010 4:02 am
Location: Mumbai, India
Thanked: 117 times
Followed by:47 members

Movie Screens in Washington County

by komal » Thu Feb 11, 2010 8:47 pm
In Washington County, attendance at the movies is just large enough for the cinema operators to make modest profits. The size of the county's population is stable and is not expected to increase much. Yet there are investors ready to double the number of movie screens in the county within five years, and they are predicting solid profits both for themselves and for the established cinema operators.

Which of the following, if true about Washington County, most helps to provide a justification for the investors'
prediction?

(A) Over the next ten years, people in their teenage years, the prime moviegoing age, will be a rapidly growing
proportion of the county's population.

(8) As distinct from the existing cinemas, most of the cinemas being planned would be located in downtown
areas, in hopes of stimulating an economic revitalization of those areas.

(C) Spending on video purchases, as well as spending on video rentals, has been increasing modestly each
year for the past ten years.

(0) The average number of screens per cinema is lower among existing cinemas than it is among cinemas still
in the planning stages.

(E) The sale of snacks and drinks in cinemas accounts for a steadily growing share of most cinema operators'
profits

OA A

User avatar
Legendary Member
Posts: 2109
Joined: Sun Apr 19, 2009 10:25 pm
Location: New Jersey
Thanked: 109 times
Followed by:79 members
GMAT Score:640

by money9111 » Thu Feb 11, 2010 8:59 pm
A because it offers a suggestion as to how cinema traffic could increase with a steady population... thus increasing profits
My goal is to make MBA applicants take onus over their process.

My story from Pre-MBA to Cornell MBA - New Post in Pre-MBA blog

Me featured on Poets & Quants

Free Book for MBA Applicants


Master | Next Rank: 500 Posts
Posts: 221
Joined: Mon Dec 22, 2008 6:24 pm
Thanked: 2 times

by mgmt_gmat » Fri Feb 12, 2010 3:23 am
IMO (E)..

because A states the increase in population without giving any reason.

Master | Next Rank: 500 Posts
Posts: 113
Joined: Thu Feb 26, 2009 8:13 am
Location: New Jersey
GMAT Score:650

by KICKGMATASS123 » Fri Feb 12, 2010 10:35 am
mgmt_gmat wrote:IMO (E)..

because A states the increase in population without giving any reason.
IMO E as well. They state that the population increases but then is the population guarenteed to go to the movies to account for increase in revenue?

GMAT Instructor
Posts: 1578
Joined: Thu May 28, 2009 8:02 am
Thanked: 128 times
Followed by:34 members
GMAT Score:760

by Osirus@VeritasPrep » Fri Feb 12, 2010 10:40 am
A- Correct, it is a logical assumption to assume that if there are more people that fall into the demographic most likely to go to the movies then that would result in more profits for the theatre

B- This has nothing to do with anything

C- This would weaken the conclusion instead of strengthening it

D- This is irrelevant. This addresses neither the expectations for increased profits nor the expectation that the population will remain steady

E- Snacks have nothing to do with the investors expecting adding more screens to the cinema to be profitable. In order for this to be the correct answer, the stimulus would have had to say something like "the investors are going to invest in a variety of new high priced snacks that have been successful in other counties".
https://www.beatthegmat.com/the-retake-o ... 51414.html

Brandon Dorsey
GMAT Instructor
Veritas Prep

Buy any Veritas Prep book(s) and receive access to 5 Practice Cats for free! Learn More.

User avatar
Legendary Member
Posts: 2109
Joined: Sun Apr 19, 2009 10:25 pm
Location: New Jersey
Thanked: 109 times
Followed by:79 members
GMAT Score:640

by money9111 » Fri Feb 12, 2010 6:14 pm
it's asking "if statement ___ were true... what would justify the prediction?" only A accomplishes this
My goal is to make MBA applicants take onus over their process.

My story from Pre-MBA to Cornell MBA - New Post in Pre-MBA blog

Me featured on Poets & Quants

Free Book for MBA Applicants


Master | Next Rank: 500 Posts
Posts: 173
Joined: Tue Jul 07, 2009 11:18 pm
Location: Hyderabad
Thanked: 12 times

by vijay_venky » Mon Feb 15, 2010 9:45 am
One more for A
And it also addresses the profits of the established cinemas predicted by the investors, so easily the best of the lot.

Master | Next Rank: 500 Posts
Posts: 140
Joined: Fri Feb 05, 2010 2:43 pm
Thanked: 3 times
GMAT Score:720

by analyst218 » Wed Feb 17, 2010 11:00 am
E is out of scope.
only A explains how the increase in revenue would be possible with a steady population.
The only explanation would be change in the proportion of population.
5 out of 10 go to movies.
in next few yrs, 9 out of 10 will go to movies. --> increased revenue.

Master | Next Rank: 500 Posts
Posts: 379
Joined: Wed Apr 07, 2010 12:53 am
Location: Chennai,India
Thanked: 3 times

by paddle_sweep » Fri Apr 30, 2010 11:45 pm
[spoiler]'A'[/spoiler], since the qn indicates a 'solid profit' for established cinemas also.

Newbie | Next Rank: 10 Posts
Posts: 2
Joined: Sun Jan 24, 2016 6:07 pm

by Singh Sanjeev » Sun Jan 24, 2016 6:10 pm
Osirus@VeritasPrep wrote:A- Correct, it is a logical assumption to assume that if there are more people that fall into the demographic most likely to go to the movies then that would result in more profits for the theatre

B- This has nothing to do with anything

C- This would weaken the conclusion instead of strengthening it

D- This is irrelevant. This addresses neither the expectations for increased profits nor the expectation that the population will remain steady

E- Snacks have nothing to do with the investors expecting adding more screens to the cinema to be profitable. In order for this to be the correct answer, the stimulus would have had to say something like "the investors are going to invest in a variety of new high priced snacks that have been successful in other counties".
Hi What is the reason that option C is a weakener. In my understanding, since video games are becoming costly, people will turn to cinema for entertainment.
Please explain.

Newbie | Next Rank: 10 Posts
Posts: 2
Joined: Sat Feb 20, 2016 10:06 am

by Sanghamitra M » Fri Feb 26, 2016 3:04 am
answer is A. Even though population is not increasing, option A saying population of movie going crowd is increasing.